problema semplice semplice

Numeri interi, razionali, divisibilità, equazioni diofantee, ...
Rispondi
superceccoWcopernico
Messaggi: 5
Iscritto il: 30 mar 2006, 16:59
Località: Brescia
Contatta:

problema semplice semplice

Messaggio da superceccoWcopernico »

Mostrare che 41 nn può essere espresso come differenza di una potenza di 2 e di una potenza di 3, cioè che non può sussistere nessuna delle due uguaglianze seguenti:
41=2^n-3^m, 41=3^n-2^m
con n,m interi positivi.

Sarei interessate a vedere qualche soluzione diversa dalla mia che pubblicherò fra qualche giorno thanks.

:twisted:
Supermegasheva
snagg
Messaggi: 70
Iscritto il: 14 mar 2005, 19:38
Contatta:

Messaggio da snagg »

EDIT: vero, scusate. Grazie Alex
Ultima modifica di snagg il 04 giu 2006, 15:38, modificato 1 volta in totale.
Alex89
Messaggi: 366
Iscritto il: 29 gen 2006, 16:57

Messaggio da Alex89 »

snagg ha scritto:Dobbiamo dimostrare che $ 41 + 3^m = 2^n $ o $ 41 + 2^m = 3^n $. Proviamo a vedere i resti mod 2 nel primo caso ora qualunque resto possano dare 41 e tutte le potenze di 3 non potranno mai fare zero. Stesso discorso vale per la seconda equazione ma mod 3. Spero di non aver aver sbagliato troppo
3^m=1 (mod.2)
41=1 (mod.2)

41+3^m=2^n
1+1=0 (mod.2)
Alex89
Messaggi: 366
Iscritto il: 29 gen 2006, 16:57

Messaggio da Alex89 »

Divido la dimostrazione in due parti:
1)Dimostro che per ogni n e m intero positivo non si avrà mai 41=2^m-3^n.

Abbiamo che 2^m>41, da cui m>5.

Sappiamo che per ogni m>3 si avrà che 2^m=0 (mod.8 ), quindi nei casi che ci riguardano avremo che 2^m=0 (mod.8 ).

Invece per quanro riguarda 3^n avrò:
per n dispari si avrà che 3^n=3 (mod.8 )
per n pari si avrà 3^n=1 (mod.8 ).

Poichè 41=1 (mod.8 ) si avrà:
1.1)2^m-3^n=41 per m>5 e n dispari: 0-3=1 (mod.8 ) impossibile.
1.2)2^m-3^n=41 per m>5 e n pari: 0-1=1 (mod.8 ) impossibile.

2)Dimostro che per ogni m e n intero positivo non si avrà mai 41=3^m-2^n.
Mod.8
Stessi casi di prima, solo 2 accettabili:
2.1)m dispari e n=1, impossibile perchè:
3^m-2^1=41
3^m=43 impossibile per m intero positivo.
2.2)m pari e n>2.
Work in progress.
piever
Messaggi: 645
Iscritto il: 18 feb 2006, 13:15
Località: Roma
Contatta:

Messaggio da piever »

Alex89 ha scritto:2.2)m pari e n>2.
Work in progress.
Soluzione:

m è l'esponente di 3 ed è pari, quindi in mod. 15 abbiamo che è congruo a 9 o a 6, per cui servirebbe che 2^n=13 oppure 10 (mod 15) ma le potenze di 2 in mod 15 sono 2, 4, 8 o 1. Quindi si verifica l'impossibilità modulo 15.
"Sei la Barbara della situazione!" (Tap)
Avatar utente
Ani-sama
Messaggi: 418
Iscritto il: 19 feb 2006, 21:38
Località: Piacenza
Contatta:

Messaggio da Ani-sama »

Allora... vediamo se può risultare accettabile...

Dividiamo la dimostrazione in 2 parti...

1) $ $2^n - 3^m=41$ $ non ha soluzioni. Non è difficile provarlo, basta vedere i resti modulo 8... i residui per le potenze di $ $2$ $ sono $ $\{2,4,0,0 \cdots\}$ $ (per esponenti di 2 crescenti); quelli delle potenze di 3 sono $ $\{3,1\}$ $, in ripetizione ciclica; Si ha che $ $41 \equiv 1 \pmod 8$ $; le uniche coppie $ (n,m) $ che rispetterebbero le congruenze sarebbero $ $(2,2k+1)$ $ e $ $(1, 2k)$ $, ma $ $2^2 - 3^{2k+1} < 0 $ $ e $ $2^1 - 3^{2k} < 0$ $, assurdo.

2) $ $3^n - 2^m = 41$ $ non ha soluzioni.

Dunque, sappiamo che $ $41 \equiv 2 \pmod 3$ $. Le classi di resto per le potenze di 3 e di 2 modulo 3 sono rispettivamente $ $\{0,0 \cdots\}$ $ e $ $\{2,1\}$ $ (in ripetizione ciclica). È dunque chiaro che per le potenze dispari di $ $2$ $ la tesi è dimostrata.

Ora ragioniamo modulo 4. Abbiamo che $ $41 \equiv 1 \pmod 4$ $. Le classi di resto per le potenze di 3 e 2 modulo 4 sono, rispettivamente, $ $\{3,1\}$ $ in ripetizione ciclica, e $ $\{2,0,0, \cdots\}$ $. Trascurando il caso $ $n=m=1$ $, similmente a prima notiamo come la tesi sia sempre verificata per le potenze dispari di $ $3$ $.

In base a quanto provato poco sopra, ci resta solo da provare la tesi per le potenze pari di $ $2$ $ e $ $3$ $, cioè, ponendo $ $n=2k, m=2k$ $:

$ ${(3^k)}^2-{(2^h)}^2=41$ $

Ora scomponiamo:

$ $(3^k-2^h)(3^k+2^h)=41$ $

La cosa che ci viene incontro è il fatto che $ $41$ $ sia un numero primo! Questo implica che uno dei due fattori che abbiamo ottenuto nella scomposizione sia uguale ad $ $1$ $. Il secondo chiaramente non può esserlo dal momento che è la somma di due numeri sicuramente positivi; occupiamoci dunque del primo fattore:

$ $3^k-2^h = 1$ $

Studiamo le classi di resto modulo 5 per le potenze di 3 e di 2... esse sono, rispettivamente, $ $\{3,4,2,1,3,4,2 \cdots\}$ $ e $ $\{2,4,3,1,2,4,3 \cdots\}$ $. Siccome $ $ 1 \equiv 1 \pmod 5$ $, notiamo che la condizione viene rispettata in certi casi (ciclici) quando $ $k=h$ $ oppure in certi altri casi (sempre ciclici) quando $ $k=h-1$ $. Studiamo una per una ciascuna di queste situazioni:

a) $ $3^h-2^h =1$ $. Essa si verifica se e solo se $ $h=1$ $, per i valori successivi la differenza aumenta visto che la funzione $ $3^x$ $ cresce più rapidamente di $ 2^x $ (non lo dimostro perché non ne ho voglia). Chiaramente l'unico valore trovato non è accettabile, in quanto, ricordando l'equazione iniziale, condurrebbe a $ $3^2-2^2 = 5 \neq 41 $.

b) $ $3^{h-1} - 2^h =1$ $. Essa si verifica se e solo se $ $h=3, k=2 $. Infatti si può dimostrare (una banale induzione o anche le derivate, come vi piace) che $ $3^{x-1}$ $ cresce più rapidamente di $ $2^x$ $. Inoltre i valori trovati non sono accettabili, visto che conducono a $ $3^4 - 2^6 = 17 \neq 41 $.

Dunque, in base a tutto quanto detto, la tesi risulta dimostrata.


P.S.

Probabilmente sono idiota io e non ho visto una soluzione banalissima, ma il problema non mi è sembrato poi quel "facile"...

EDIT: ho visto la soluzione di piever. Sì, sono idiota... :?
...
superceccoWcopernico
Messaggi: 5
Iscritto il: 30 mar 2006, 16:59
Località: Brescia
Contatta:

re anisama:

Messaggio da superceccoWcopernico »

Complimenti per le soluzioni proposte comunque io l'ho fatto così anche se nn è la più corta mi sembra giusta e rigorosa:

proviamo che non è mai verificata 41=3^n-2^m.
Supponiamo per assurdo che esistano due interi n,m per cui l'uguaglianza vale; prendendo i resti modulo 3 del primo e del secondo membro si ha :
41 mod 3 =2= -2^m mod 3
e -2^m mod 3 vale 2 per m pari e 1 per m dispari; quindi m deve essere pari,
m= 2k, e dunque esistono n,k tali che 41= 3^n-2^(2k).
Passando ai resti modulo 4 abbiamo:
41 mod 4=1=3^n mod 4
e 3^n mod 4 vale 1 per n pari e 3 per n dispari; quindi n deve essere pari, n=2h, ed esistono h, k tali che:
41= 2^(2h)-3^(2k)=(3^h+2^k)(3^h-2^k); dato che 41 è primo, dovrebbero aversi che 3^h-2^k=1, e quindi:
41= (2^k+1)+2^k=2^(k+1)+1 e questo è impossibile perchè 40 non è una potenza di 2.
Proviamo ora che non è mai verificata 41= 2^n-3^m; se per assurdo vi fossero due interi
n.m per cui l'uguaglianza vale, prendendo i resti modulo 3 a primo e secondo membro dovremmo avere che:
41 mod 3=2=2^n mod 3;
L'espressione 2^n mod 3 vale 1 per n pari e 2 per n dispari e quindi n deve essere dispari,
n=2h+1; esistono pertanto h,m tali che 41=2^(2h+1)-3^m.
Passando ai resti modulo 4 abbiamo:
41 mod 4=1=-3^m mod 4,
ma -3^m mod 4 vale 3 per m pari e 1 per m dispari, e quindi m deve essere dispari, m=2k+1, ed
esistono h,k tali che 41=2^(2h+1)-3^(2k+1)=2*4^h-3*9^k; questo però è impossibile, perchè passando ai resti
modulo 8 avremmo che 41 mod 8=1, mentre:
(2*4^h-3*9^k)mod8 = -3(8+1)^k mod8=5

:twisted:
Supermegasheva
Rispondi